K
Khách

Hãy nhập câu hỏi của bạn vào đây, nếu là tài khoản VIP, bạn sẽ được ưu tiên trả lời.

16 tháng 10 2020

Ta có: \(2a^2+\frac{b^2}{4}+\frac{1}{a^2}=4\Rightarrow8a^4+a^2b^2+4=16a^2\Rightarrow a^2b^2=-8a^4+16a^2-4=-8\left(a^4-2a^2+1\right)+4=-8\left(a^2-1\right)^2+4\le4\)\(\Rightarrow\left|ab\right|\le2\Rightarrow-2\le ab\le2\)

Vậy MaxS = 2023 khi ab = 2 và a2 = 1 do đó \(\left(a,b\right)\in\left\{\left(-1;-2\right);\left(1;2\right)\right\}\)

MinS = 2019 khi ab = -2 và a2 = 1 do đó \(\left(a,b\right)\in\left\{\left(-1;2\right);\left(1;-2\right)\right\}\)

14 tháng 8 2017

a^2 hay a.2 thế

14 tháng 8 2017

a^2 bn ạ!!
 

29 tháng 4 2018

We have : \(A=\frac{1}{a^2+b^2}+\frac{1}{2ab}+\frac{1}{2ab}\)

By Cauchy - Schwarz and AM - GM have :

\(A\ge\frac{\left(1+1\right)^2}{a^2+b^2+2ab}+\frac{1}{2.\frac{\left(a+b\right)^2}{4}}=\frac{4}{\left(a+b\right)^2}+\frac{2}{\left(a+b\right)^2}=\frac{6}{\left(a+b\right)^2}\ge6\)

Then greatest posible of A is 6 when \(a=b=\frac{1}{2}\)

NV
1 tháng 11 2021

\(S=\dfrac{1}{a^3+b^3}+\dfrac{1}{a^2b}+\dfrac{1}{ab^2}\ge\dfrac{1}{a^3+b^3}+\dfrac{4}{a^2b+ab^2}\)

\(S\ge\left(\dfrac{1}{a^3+b^3}+\dfrac{1}{a^2b+ab^2}+\dfrac{1}{a^2b+ab^2}+\dfrac{1}{a^2b+ab^2}\right)+\dfrac{1}{ab\left(a+b\right)}\)

\(S\ge\dfrac{16}{a^3+b^3+3a^2b+3ab^2}+\dfrac{1}{\dfrac{\left(a+b\right)^2}{4}.\left(a+b\right)}=\dfrac{20}{\left(a+b\right)^3}\ge20\)

\(S_{min}=20\) khi \(a=b=\dfrac{1}{2}\)

11 tháng 12 2016

Đầu tiên ta chứng minh bổ đề. 

Ta có

\(6=3.\frac{a^2}{3}+2.\frac{b^2}{2}+c^2\)

\(\ge6.\sqrt[6]{\left(\frac{a^2}{3}\right)^3.\left(\frac{b^2}{2}\right)^2.c^2}=6.\sqrt[6]{\frac{a^6b^4c^2}{3^3.2^2}}\)

\(\Rightarrow a^6b^4c^2\le3^3.2^2\)

Ta lại có:

\(P=3.\frac{a}{3bc}+4.\frac{b}{2ca}+5.\frac{c}{ab}\)

\(\ge12.\sqrt[12]{\left(\frac{a}{3bc}\right)^3.\left(\frac{b}{2ca}\right)^4.\left(\frac{c}{ab}\right)^5}\)

\(=\frac{12}{\sqrt[12]{3^3.2^4}.\sqrt[12]{a^6b^4c^2}}\)

\(\ge\frac{12}{\sqrt[12]{3^3.2^4}.\sqrt[12]{3^3.2^2}}=2\sqrt{6}\)

Dấu = xảy ra khi \(\hept{\begin{cases}a=\sqrt{3}\\b=\sqrt{2}\\c=1\end{cases}}\)

28 tháng 5 2018

\(A\ge\frac{9}{a+2+b+2+c+2}+\frac{1}{9abc}\)

\(\Rightarrow A\ge\frac{9}{7}+\frac{1}{9abc}\)

Theo BĐT AM-GM ta có: \(1=a+b+c\ge3\sqrt[3]{abc}\)

\(\Rightarrow abc\le\frac{1}{27}\)

\(\Rightarrow\frac{1}{9abc}\ge3\)

Do đó ta có: 

\(A\ge\frac{9}{7}+3=\frac{30}{7}\)